LSAT and Law School Admissions Forum

Get expert LSAT preparation and law school admissions advice from PowerScore Test Preparation.

 Administrator
PowerScore Staff
  • PowerScore Staff
  • Posts: 8916
  • Joined: Feb 02, 2011
|
#22797
Complete Question Explanation

Weaken. The correct answer choice is (E)

In this dialogue, Rita makes the argument that beyond a certain point, payments to farmers are not in line with the original intent of the subsidies. The crux of Thomas' argument is that such a cutoff would be impossible to implement because the subsidies are based on future incomes that cannot be accurately predicted. To weaken Thomas' argument, we might look for an answer choice that would allow for implementation of Rita's suggested cutoff.

Answer choice (A): The accessibility of bank loans to farmers has no effect on Thomas' objection, so this answer choice is incorrect.

Answer choice (B): This answer choice deals with the ramifications of implementing such cutoff points—this is not the basis of Thomas' objection. Since this answer makes Thomas' argument no stronger or weaker, this answer choice is incorrect.

Answer choice (C): We might not be surprised that farmers have different sales from year to year, and this does not weaken Thomas' argument about the impossibility of imposing cutoffs.

Answer choice (D): Like incorrect answer choice (B) above, this wrong answer deals with the ramifications of cutoffs once they are implemented. Thomas objects based on what he perceives as the impossibility of implementation.

Answer choice (E): This is the correct answer choice, as it provides a simple solution to Thomas' problem with implementing the cutoffs, as prephrased above.

Get the most out of your LSAT Prep Plus subscription.

Analyze and track your performance with our Testing and Analytics Package.